Difference between revisions of "2002 AMC 10B Problems/Problem 23"

(Solution 3)
(Solution 4)
Line 26: Line 26:
  
 
==Solution 4==
 
==Solution 4==
 +
 +
We can set <math>n</math> equal to <math>m</math>, so we can say that
 +
<cmath>a_{m + m} = a_m + a_m = m*m</cmath>
 +
<cmath>a_{2m} = 2a_m + m^2</cmath>
 +
 +
We set <math>2m = 12</math>, we get <math>m = 6</math>.
 +
<cmath>a_{12} = 2a_6 + 36</cmath>
 +
 +
We set <math>2m = 6</math>m, we get <math>m = 3</math>.
 +
<cmath>a_6 = 2a_3 + 9</cmath>
 +
 +
Solving for <math>a_3</math> is easy, just direct substitution.
 +
<cmath>a_2 = 1 + 1 + 1 = 3</cmath>
 +
<cmath>a_3 = a_{2 + 1} = 3 + 1 + 2 = 6</cmath>
 +
 +
Substituting, we get
 +
<cmath>a_6 = 2(6) + 9 = 21</cmath>
 +
<cmath>a_12 = 2(21) + 36 = 78</cmath>
 +
 +
Thus, the answer is <math>\boxed{D}</math>.
  
 
== Additional Comment==
 
== Additional Comment==

Revision as of 17:04, 27 December 2019

Problem 23

Let $\{a_k\}$ be a sequence of integers such that $a_1=1$ and $a_{m+n}=a_m+a_n+mn,$ for all positive integers $m$ and $n.$ Then $a_{12}$ is

$\mathrm{(A) \ } 45\qquad \mathrm{(B) \ } 56\qquad \mathrm{(C) \ } 67\qquad \mathrm{(D) \ } 78\qquad \mathrm{(E) \ } 89$

Solution 1

When $m=1$, $a_{n+1}=1+a_n+n$. Hence, \[a_{2}=1+a_1+2\] \[a_{3}=1+a_2+3\] \[a_{4}=1+a_3+4\] \[\dots\] \[a_{12}=1+a_{11}+11\] Adding these equations up, we have that $a_{12}=12+(1+2+3+...+11)=\boxed{78}$

~AopsUser101

Solution 2

Substituting $n=1$ into $a_{m+n}=a_m+a_n+mn$: $a_{m+1}=a_m+a_{1}+m$. Since $a_1 = 1$, $a_{m+1}=a_m+m+1$. Therefore, $a_m = a_{m-1} + m, a_{m-1}=a_{m-2}+(m-1), a_{m-2} = a_{m-3} + (m-2)$, and so on until $a_2 = a_1 + 2$. Adding the Left Hand Sides of all of these equations gives $a_m + a_{m-1} + a_{m-2} + a_{m-3} + \cdots + a_2$; adding the Right Hand Sides of these equations gives $(a_{m-1} + a_{m-2} + a_{m-3} + \cdots + a_1) + (m + (m-1) + (m-2) + \cdots + 2)$. These two expressions must be equal; hence $a_m + a_{m-1} + a_{m-2} + a_{m-3} + \cdots + a_2 = (a_{m-1} + a_{m-2} + a_{m-3} + \cdots + a_1) + (m + (m-1) + (m-2) + \cdots + 2)$ and $a_m = a_1 + (m + (m-1) + (m-2) + \cdots + 2)$. Substituting $a_1 = 1$: $a_m = 1 + (m + (m-1) + (m-2) + \cdots + 2) = 1+2+3+4+ \cdots +m = \frac{(m+1)(m)}{2}$. Thus we have a general formula for $a_m$ and substituting $m=12$: $a_{12} = \frac{(13)(12)}{2} = (13)(6) = \boxed{\mathrm{(D) \ } 78}$.

Solution 3

We can literally just plug stuff in. No prerequisite is actually said in the sequence. Since $a_{m+n} = a_m+a_n +mn$, we know $a_2=a_1+a_1+1\cdot1=1+1+1=3$. After this, we can use $a_2$ to find $a_4$. $a_4=a_2+a_2+2\cdot 2 = 3+3+4 = 10$. Now, we can use $a_2$ and $a_4$ to find $a_6$, or $a_6=a_4+a_2+4\cdot 2 = 10+3+8=21$. Lastly, we can use $a_6$ to find $a_{12}$. $a_{12} = a_6+a_6+6\cdot 6 = 21+21+36= \boxed{(D) 78}$

Solution 4

We can set $n$ equal to $m$, so we can say that \[a_{m + m} = a_m + a_m = m*m\] \[a_{2m} = 2a_m + m^2\]

We set $2m = 12$, we get $m = 6$. \[a_{12} = 2a_6 + 36\]

We set $2m = 6$m, we get $m = 3$. \[a_6 = 2a_3 + 9\]

Solving for $a_3$ is easy, just direct substitution. \[a_2 = 1 + 1 + 1 = 3\] \[a_3 = a_{2 + 1} = 3 + 1 + 2 = 6\]

Substituting, we get \[a_6 = 2(6) + 9 = 21\] \[a_12 = 2(21) + 36 = 78\]

Thus, the answer is $\boxed{D}$.

Additional Comment

This is also the formula for the triangular numbers $T_n=1+2+3+...+n$, as seen in Solution 2

See also

2002 AMC 10B (ProblemsAnswer KeyResources)
Preceded by
Problem 22
Followed by
Problem 24
1 2 3 4 5 6 7 8 9 10 11 12 13 14 15 16 17 18 19 20 21 22 23 24 25
All AMC 10 Problems and Solutions

The problems on this page are copyrighted by the Mathematical Association of America's American Mathematics Competitions. AMC logo.png